Consider the following function. Complete parts (a) through (e) below.f(x)=x²-2x-8The vertex is.(Type an ordered pair.)c. Find the x-intercepts. The x-intercept(s) is/are(Type an integer or a fraction. Use a comma to separate answers as needed.)d. Find the y-intercept. The y-intercept is(Type an integer or a fraction.)e. Use the results from parts (a)-(d) to araph the quadratic function.

Answers

Answer 1

Given the function:

[tex]f(x)=x^2-2x-8[/tex]

It is a quadratic function where:

a=1

b= -2

c= -8

The x-coordinate of the vertex is given by:

[tex]x=-\frac{b}{2a}[/tex]

Substitute a and b:

[tex]x=-\frac{-2}{2(1)}=\frac{2}{2}=1[/tex]

Substituting in the original equation to obtain the y-coordinate, we obtain:

[tex]y=(1)^2-2(1)-8=1-2-8=-9[/tex]

So, the vertex is (0, -9)

c. For the intercept at x we make y = 0:

[tex]0=x^2-2x-8[/tex]

And solve for x by factorization:

[tex]\begin{gathered} (x-4)(x+2)=0 \\ Separate\text{ the solutions} \\ x-4=0 \\ x-4+4=0+4 \\ x=4 \\ and \\ x+2=0 \\ x+2-2=0-2 \\ x=-2 \end{gathered}[/tex]

So, the x-intercepts are:

(-2, 0) and (4,0)

Answer: (-2,0), (4,0)

d. For the intercept at y we make x = 0:

[tex]y=(0)^2-2(0)-8=-8[/tex]

So the y-intercept is (0, -8)

Answer: (0, -8)

e. Graphing the function:

Consider The Following Function. Complete Parts (a) Through (e) Below.f(x)=x-2x-8The Vertex Is.(Type

Related Questions

*1. If the variable x represents the total number of COVID-19 deaths in the United States since March 1,I 2020, what do the following expressions represent?a. X - 100,000

Answers

The expression represents the number of COVID deaths since March 1 2020 minus 100,000 deaths.

If the measures of the angles of a triangle arerepresented by 2x, 3x - 15, and 7x +15, the triangleis1) an isosceles triangle2) a right triangle3) an acute triangle4) an equiangular triangle

Answers

Answer

Option 1 is correct.

The triangle is an isosceles triangle.

Explanation

Noting that the sum of angles in a triangle is 180°.

We can solve for each of the angles in this triangle to obtain the type of triangle it is.

The angles of the triangle are 2x, (3x - 15) and (7x + 15)

2x + 3x - 15 + 7x + 15 = 180°

2x + 3x + 7x - 15 + 15 = 180°

12x = 180°

Divide both sides by 12

(12x/12) = (180°/12)

x = 15°

We can then solve for the measures of the three angles now

2x = 2 (15°) = 30°

3x - 15 = 3 (15°) - 15° = 45° - 15° = 30°

7x + 15 = 7 (15°) + 15° = 105° + 15° = 120°

So, the angles of the triangle are 30°, 30° and 120°

A tringle that has two of its angles equal to each other is called an isosceles triangle.

Hope this Helps!!!

Evaluate the function when x= -2,0, and 5 h(x)= -2x+9

Answers

Given:

a function is given as h(x) = -2x + 9

Find:

we have to evaluate the function at x = -2 , 0 and 5.

Explanation:

when x = -2

h(-2) = -2(-2) + 9 = 4 + 9 = 13

when x = 0

h(0) = -2(0) + 9 = 0 + 9 = 9

when x = 5

h(5) = -2(5) + 9 = - 10 + 9 = -1

Therefore, the values of given function h(x) are 13, 9 , -1 at x = -2, 0 , 5 respectively.

Find the starting value and the base for the exponential function f(x)=kb^x that passes through the two points:(0,3) and (2,12).The starting value k is: AnswerThe base b is: Answer

Answers

The exponential equation given is,

[tex]f(x)=kb^x[/tex]

Given the points

[tex](0,3)\text{ and (2,12)}[/tex]

Therefore, the values for k and b will be resolved graphically.

Let us now plot the graph using a graphical calculator

From the graph,

[tex]\begin{gathered} y_1=f(x) \\ a=k=3 \\ b=2 \end{gathered}[/tex]

Final answers

[tex]\begin{gathered} k=3 \\ b=2 \end{gathered}[/tex]

Johann uses 42 +7 to represent the number of players who are on teams.Explain what 42÷7 means. Enter a number in each box.

Answers

Johann uses 42 +7 to represent the number of players who are on teams.

Explain what 42÷7 means.

_______________________________

42 +7 = the number of players who are on teams

42 = he number of players who are on teams minus 7

42 players

________________________

Dividing by 7 (the number of player per group)

Each team has 7 players

_____________

How do I find all possible rational zeros in a polynomial function?f(x) = x^4-2x^3-4x^2+2x+3

Answers

The factors are 1 , -1 twice and 3

Here, we want to find the rational roots of;

[tex]f(x)\text{= }x^4-2x^3-4x^2+2x+3[/tex]

We can start here by trying out simple numbers if we cannot factorize the polynomial at a go

The given polynomial here can be factorized

We can express it as;

[tex]x^4-2x^3-4x^2+2x+3\text{ = (}x-1)(x+1)^2(x-3)[/tex]

So what we have here is to simply equate individual linear factor to zero

Thus, we have the factors as;

1 , -1 twice and 3

Suppose that you decide to buy a car for $32,635, including taxes and license fees. You saved $9,000 for a down payment and can get a four year car loan at 6.31%. Find the monthly payment and the total interest for the loan. Equation below.

Answers

SOLUTION:

Step 1:

In this question, we are given the following:

Suppose that you decide to buy a car for $32,635, including taxes and license fees.

You saved $9,000 for a down payment and can get a four-year car loan at 6.31%.

Find the monthly payment and the total interest for the loan.

Step 2:

In this question, we are given:

Cost of the car = $ 32, 635

Down payment = $ 9000

Principal on the Car loan = $ ( 32,635 - 9,000 ) = $ 23, 635

Rate = 6. 31%

Using the PMI Formulae, we have that:

Complete the statements about the following numbers: 2/7, 0.1, 0.9, 6/8. Use the + and - buttons to change how many ticks are displayed. The number represents the amount of even segments between 0 and 1. The point closest to the benchmark 1 is at The point closest to the benchmark O is at How would you order these fractions and decimals from least to greatest? Click or tap and drag to move the dot along the number line. 1 B 2 2 +

Answers

To solve the exercise it is easier to convert all the given points to decimals. So,

[tex]\frac{2}{7}=0.23[/tex][tex]\frac{6}{8}=0.75[/tex]

Then,

*The point closest to the benchmark 1 is at 0.9.

*The point closest to the benchmark 0 is 0.1.

*Ordering these points from least to greatest you have:

[tex]\begin{gathered} 0.1 \\ 0.23=\frac{2}{7} \\ 0.75=\frac{6}{8} \\ 0.9 \end{gathered}[/tex]

A painter has three partially filled paint cans. One contains1 7/8 gallons, the second contains1 1/5 gallons, and the third contains1 3/4 gallons. Which answer is closest to the total amount of paint?

Answers

[tex]\text{total amount=}\frac{193}{40}=4.825\text{ gallons}[/tex]

Explanation

Step 1

convet the mixed numbers into simple fractions

remember

[tex]a\frac{b}{c}=\frac{(a\cdot c)+b}{c}[/tex]

then

[tex]\begin{gathered} 1\text{ }\frac{7}{8}=\frac{(1\cdot8+7)}{8}=\frac{15}{8} \\ 1\frac{1}{5}=\frac{(1\cdot5+1)}{5}=\frac{6}{5} \\ 1\frac{3}{4}=\frac{(1\cdot4+3)}{4}=\frac{7}{4} \end{gathered}[/tex]

Step 2

now, make the sum to find the total amount

[tex]\begin{gathered} \text{total amount= }\frac{15}{8}+\frac{6}{5}+\frac{7}{4} \\ \text{total amount=}\frac{(15\cdot5\cdot4)+(6\cdot8\cdot4)+(7\cdot5\cdot8)}{160} \\ \text{total amount=}\frac{300+192+280}{160} \\ \text{total amount=}\frac{772}{160} \\ \text{total amount=}\frac{193}{40} \end{gathered}[/tex]

I hope this helps you

16. Solve this system: y = 3x+1 y= 5x-3

Answers

y=3x+1 (1)

y=5x-3 (2)

To solve this system, we can use the equalize method:

3x+1=5x-3

3+1=5x-3x

4=2x

x=4/2

x=2

Now, substitung x=2 in y=3x+1

y=3(2)+1

y=6+1

y=7

Then, the solution to this system of equation would be: (2, 7)


"Radon: The Problem No One Wants to Face" is the title of an article appearing in Consumer Reports. Radon is a gas emitted from the ground that can collect in houses and buildings. At
certain levels it can cause lung cancer. Radon concentrations are measured in picocuries per liter (PCI/L). A radon level of 4 pci/L is considered "acceptable." Radon levels in a house vary
from week to week. In one house, a sample of 8 weeks had the following readings for radon level (in pcI/L).
1.9 2.8 5.7 4.8 1.9 8.6 3.9 7.3

(a) Find the mean, median, and mode. (Round your answers to two decimal places.)
mean
median
mode

(b) Find the sample standard deviation, coefficient of variation, and range. (Round your answers to two decimal places.)
S
CV
range

(c) Based on the data, would you recommend radon mitigation in this house? Explain.
O Yes, since the average value is over "acceptable" ranges, although the median value is not.
O Yes, since the median value is over "acceptable" ranges, although the mean value is not.
O No, since the average and median values are both under "acceptable" ranges.
O Yes, since the average and median values
are both over "acceptable"
ranges.

Answers

a) The mean of the data set is 4.61

The median of the data set is 4.35

The mode of the data set is 1.9

b) Sample standard deviation is 2.58

Coefficient of Variation is 55.96

Range is 6.7

Given,

The data set;

1.9, 2.8, 5.7, 4.8, 1.9, 8.6, 3.9, 7.3

a) We have to find the mean, median and mode

Mean = (1.9 + 2.8 + 5.7 + 4.8 + 1.9 + 8.6 + 3.9 + 7.3) / 8 = 36.9/8 = 4.61

Median;

Order the data first;

1.9, 1.9, 2.8, 3.9, 4.8, 5.7, 7.3, 8.6

Now,

The data is of even number, so;

Median = [(n/2) + (n/2 + 1)] / 2

Here,

n/2 = 8/2 = 4th term

n/2 + 1 = 5th term

Then,

Median = (3.9 + 4.8) / 2 = 8.7/2 = 4.35

Mode;

The mode of the given data is 1.9

b) Sample Standard Deviation

Here it is the formula to calculate it:

x = √(∑(xi - x)²/n-1))

sₓ = √(46.85/7) ≈ 2.58

Coefficient of Variation

CV is the quotient between sample Standard deviation over Mean and it is used to make comparisons.

CV = sₓ/x × 100 = 2.58/4.61 x 100 = 55.96

Range

The difference between the highest and the lowest value of this sample

8.6 - 1.9=6.7

Learn more about data set here;

https://brainly.com/question/13708278

#SPJ1

The exit is exactly half way between the Ferris wheel and where you parked your car. Give the coordinates of your parking spot.

Answers

GIven data :

The coordinates of ferris wheel is, (2,7).

The coordinates of exit is (4,0).

they have given exactly half way so let us use the midpoint formula,

the mid point formula is,

[tex](\frac{x_1+x_2}{2},\frac{y_1,y_2}{2})\ldots(1)[/tex]

take the coordinates as

[tex]\begin{gathered} (x_1,y_1)=(2,7) \\ (x_2,y_2)=(4,0) \end{gathered}[/tex]

let us subsitute in eqiuation (1),

[tex]\begin{gathered} (\frac{2+4_{}}{2},\frac{7_{}+0_{}}{2}) \\ (\frac{6}{2},\frac{7}{2}) \\ (3,3.5) \end{gathered}[/tex]

thus the coordinates of parking spot is (3,3.5).

The results of a survey show that the percent of adults in a certain town who want to add bike lanes to amajor roadway is in the interval (0.57, 0.65) (9 points)(a) What is the point estimate for the percent who want to add the bike lanes?(b) What is the poll's margin of error?(c) If the town's adult population is 31,526, what is the best estimate for the number of people whowould support the bike lanes?

Answers

If we know the confidence interval for the proportion, the point estimate will be at the center of this interval.

Then, we can calculate the point estimate p as the average between the boundaries of the interval:

[tex]p=\frac{0.57+0.65}{2}=\frac{1.22}{2}=0.61[/tex]

The margin of error can be calculated, knowing the interval, as half the difference between the upper boundary and the lower boundary of the interval:

[tex]\text{MOE}=\frac{UB-LB}{2}=\frac{0.65-0.57}{2}=\frac{0.08}{2}=0.04[/tex]

The margin of error is 0.04. This margin of error is also the absolute difference between any boundary of the interval and the point estimate.

If the town's population is 31,526, the best estimate for the number of people who

would support the bike lanes is to use the point estimate as the proportion:

[tex]X=p\cdot N=0.61\cdot31526\approx19231[/tex]

Answer:

a) The point estimate is p=0.61

b) The margin of error is MOE = 0.04

c) The best estimate is X=19231

The hotel room tax in Junction City is calculated using the function T(x) =0.06x + 4.5, where x = the cost in dollars of the room and T(x) = the tax in dollars. What is the tax on a room that costs $120?

Answers

Since the tax on the room is given by the function:

T(x)=0.06x+45

And x(cost in dollars of the room) is given= $120, you just have to subtitue x=120 on the function

T(120)=0.06(120)+4.5

T(120)=7.2+4.5

T(120)=11.7

So, the tax in dollars would be $11.7

hi my name is Shila and I'm trying to explain how to solve this problem to my daughter but a but confused can you direct me please?

Answers

As given by the question

There are given that the point

[tex]\frac{8}{6}[/tex]

Now,

First, break the point

So,

[tex]\frac{8}{6}=\frac{4}{3}[/tex]

According to the above point, there are showing 3 in the denominator

So, between 0 and 1 divide 3 parts in the number line

Then,

A train travels 165 km in 1.5 hours.
How far will the train travel in 2.2 hours if it maintains the same speed?

Answers

Hello There, today we will be solving your problem

[tex]\huge\red{\mid{\underline{\overline{\textbf{EQUATION AND ANSWER}}}\mid}}[/tex]

_________________

Let's solve this equation using rates,

_________________Definitions

Unit Rate - A unit rate means a rate for one of something.

Cross Multiplication - In mathematics, specifically in elementary arithmetic and elementary algebra, given an equation between two fractions or rational expressions, one can cross-multiply to simplify the equation or determine the value of a variable.

_________________

Now that we understand the definition we can further solve this equation

[tex]\large\red{\mid{\underline{\overline{\textbf{Values}}}\mid}}[/tex]

[tex]165[/tex] [tex]km[/tex] ⇒ [tex]1.5[/tex] [tex]hr[/tex]

[tex]x\\[/tex] [tex]km[/tex] ⇒ [tex]2.2[/tex] [tex]hr[/tex]

Now we will use cross-multiplication to solve this equation

[tex]\large\red{\mid{\underline{\overline{\textbf{Equtation}}}\mid}}[/tex]

[tex]165 \cdot 2.2\\x\cdot1.5[/tex]

Once solving this equation we get

[tex]1.5x=363[/tex] [tex]km[/tex]

Divide both sides by [tex]1.5[/tex]

[tex]x=242[/tex] [tex]km[/tex]

[tex]\large\red{\mid{\underline{\overline{\textbf{Answer}}}\mid}}[/tex]

A train travels 165 km in 1.5 hours. How far will the train travel in 2.2 hours if it maintains the same speed?

The train would've traveled a total of 242 km in 2.2 hours.

Have a good day!

Find the measure of a positive angle and a negative angle that are coterminal with 100° sketch of three angles labeling clearly with directional arrows.

Answers

Coterminal angles are different angles that have the same terminal side.

A positive angle has one turn more around so it has a measure of 100°+360° = 460°.

A negative angle will have a measure that is represented in clockwise rotation and be equal to 100° - 360° = -260°.

We can sketch an angle of measure 100°, a positive coterminal angle and a negative coterminal angle as:

please help me!!!!!!

Answers

I attach the table with the results organized correctly.

The correct option is 1.

Kathy wants to buy a condominium selling for $96,000. The taxes on the property are $1300 per year, and homeowners' insurance is $336 per year. Kathy's gross monthly income is $4000. She haher van. The bank is requiring 20% down and is charging a 9.5% interest rate with no points. Her bank will approve a loan that has a total monthly mortgage payment of principal, interest, property tthan or equal to 28% of her adjusted monthly income. Complete parts a) through h) below.a) Determine the required down payment.The required down payment is $b) Determine 28% of her adjusted monthly income.28% of her adjusted monthly income is $(Round to the nearest cent as needed.)c) Determine the monthly payments of principal and interest for a 25-year loan.The monthly payment of principal and interest for a 25-year loan is $(Round to the nearest cent as needed.)d) Determine her monthly payment, including homeowners' insurance and taxes.Her total monthly payment, including homeowners' insurance and taxes is $(Round to the nearest cent needed.) Does Kathy qualify for the loan?0 YesO No

Answers

a) the cost of the house is 96000 and the down paidment is the 20% so we can use a rule of 3 to solve it so:

[tex]\begin{gathered} 96000\to100 \\ x\to20 \end{gathered}[/tex]

so the equation will be:

[tex]\begin{gathered} x=\frac{96000\cdot20}{100} \\ x=19200 \end{gathered}[/tex]

b) her income is $4000 so the 28% will be:

[tex]\begin{gathered} 4000\to100 \\ x\to28 \end{gathered}[/tex]

so the equation will be:

[tex]\begin{gathered} x=\frac{4000\cdot28}{100} \\ x=1120 \end{gathered}[/tex]

c) the equation that models a loan is:

[tex]C=\frac{P\cdot(0.095\cdot(1+0.095)^n)}{(1+0.095)^{25}-1}[/tex]

So we replace the princeiple and find monthly paidment.

[tex]\begin{gathered} C=\frac{76800\cdot(0.095\cdot(1.095)^{25}}{8.67} \\ C=\frac{76800\cdot0.92}{8.67} \\ C=\frac{70540.30}{8.67} \\ C=8136.14 \\ C\approx8136 \end{gathered}[/tex]

d)The total monthly paidment will be:

[tex]\begin{gathered} T=8136+\frac{1300}{12}+\frac{336}{12} \\ T=8136+108.33+28 \\ T=8272.33 \\ T\approx8272 \end{gathered}[/tex]

SOo the answer is NO she can't afort to buy this house

Convert percent to decimal 51.2% =

Answers

Let's begin by identifying key information given to us:

51.2% = 51.2/100

[tex]\begin{gathered} 51.2\text{ \%}=\frac{51.2}{100} \\ \Rightarrow0.512 \end{gathered}[/tex]

Add Solve: n + 7 = 31

Answers

Answer:

n = 24

Explanation:

The initial expression is:

n + 7 = 31

So, to solve the equation, we need to subtract 7 from both sides:

n + 7 - 7 = 31 - 7

n = 24

Therefore, the solution is n = 24

If 8 more than twice a number is 14, what is 8 times the number?

Answers

Let the unkonwn number be

[tex]=x[/tex]

Twice the number means

[tex]\begin{gathered} =2\times x \\ =2x \end{gathered}[/tex]

8 more than twice the number means the sum of twice the number and 8

[tex]=2x+8[/tex]

8 more than twice a number is 14, will be represented below as

[tex]2x+8=14[/tex]

collect similar terms from the equation above to get

[tex]\begin{gathered} 2x+8=14 \\ 2x=14-8 \\ 2x=6 \\ \text{divide both sides by 2} \\ \frac{2x}{2}=\frac{6}{2} \\ x=3 \end{gathered}[/tex]

8 times the number will then be,

[tex]\begin{gathered} =8\times x \\ =8\times3 \\ =24 \end{gathered}[/tex]

Hence,

The correct answer is OPTION C

Linda is adding padding to all of the surfaces inside her attic for extra warmth in the winter.She needs to find the approximate surface area of the attic, including the walls, floor, andceiling. The attic is in the shape of a triangular prism. Linda draws the net and writesthe expression below to represent the surface area of the attic. Are Linda's net andexpression correct?15 ft45 ft25 ft40 ft25 ft25 ft25 ft- 15 ft45 ft40 ft15 ftExpression for Surface Area of Attic:45 (40 + 25 + 25) + ] (40 x 15)

Answers

We can formulate an expression for the surface area of the attic like this:

The area of a triangle is given by the following formula:

[tex]A=\frac{b\times h}{2}[/tex]

Where b is the base and h is the height of the triangle.

The area of a rectangle is given by the following formula:

[tex]A=w\times l[/tex]

Where w is the width and l is the length of the rectangle.

In this case, the attic has three rectangular faces, all of them have a width of 45 ft. two of them have a length of 25 ft and one has a width of 40 ft, then we can calculate the areas of these faces like this:

[tex]\begin{gathered} A1=45\times40 \\ A2=45\times25 \\ A3=45\times25 \end{gathered}[/tex]

By summing up these areas, we get the area of the rectangular faces:

[tex]A=45\times40+45\times25+45\times25[/tex]

From this expression, we can factor 45 to get:

[tex]A=45\times(40+25+25)[/tex]

For the two triangular faces, their height equals 15 ft and the length of the bases equals 40 ft, then their areas are:

[tex]\begin{gathered} A1=\frac{15\times40}{2} \\ A2=\frac{15\times40}{2} \end{gathered}[/tex]

By summing them up, we get the area of the triangular faces:

[tex]A=\frac{15\times40}{2}+\frac{15\times40}{2}=15\times40[/tex]

By summing the area of the rectangular faces and the area of the triangular faces, we get the expression to calculate the total surface area of the attic, like this:

[tex]A=45(40+25+25)+40\times15=4650[/tex]

Then, the net Linda draw is correct. The first term of Linda's expression 45(40+25+25) is correct. The second term of Linda's equation missing a factor of 2. The surface area of Linda's attic is 4650 square feet

If a person travels 3.5 miles in 30 minutes, what is their speed i miles per hour

Answers

Given:-

If a person travels 3.5 miles in 30 minutes.

To find their speed in miles per hour.

So now we solve using the formula,

[tex]\text{Distance}=\text{speed}\times time[/tex]

Substituting the known values. we get,

[tex]3.5=\text{Speed}\times\frac{1}{2}[/tex]

Now we solve for speed. so we get,

[tex]\begin{gathered} \text{speed}=3.5\times2 \\ \text{speed}=7 \end{gathered}[/tex]

So the required speed is 7miles/hr.

3.2 Hangalakani works as a builder for 6,5 hours per day excluding 30 minutes tea break 1 hour lunch. He starts working at 07:30 am 2.1 Determine the time of the day Hangalakani leaves work for home 312.2 Hangalakani calculated that 245,37 bags of cement are required for the job His manager states that they need to purchase 246 bags Explain why the manager's statement is correct.

Answers

Hangalakani leaves his work for home at 3.30 PM .

Hangalakani starts his work at 7.30 am.

He works for 6.5 hours.

He then takes a lunch break for 1 hour .

He then takes tea break for about 30 minutes = 0.5  hour

Total time spent at work = 6.5 + 1 + 0. 5 = 8 hours.

So if he starts work at 7.30 then he will end at

7.30 am + 8 hours  = 3.30 pm in the afternoon .

Now the manager says that they need 246 bags while Hangalakani calculated they need 245.37 bags of cement.

The manager is correct because bags of cement are not sold as a part or decimal .  The number of bags that can be bought must be a whole number.

To learn more about work visit:

https://brainly.com/question/28265361

#SPJ9

A and B are mutually exclusive events P(A) =0.60 and P(B)=0.30 what is P (A or B)

Answers

We know that for any number of mutually exclusive events, we have the formula:

[tex]P(A_1\cup A_2\cup A_3\cup\ldots)=P(A_1)+P(A_2)+P(A_3)+\cdots[/tex]

In this case, we have that P(A)=0.60 and P(B)=0.30, then:

[tex]\begin{gathered} P(A\cup B)=P(A)+P(B)=0.60+0.30=0.90 \\ P(A\cup B)=0.90 \end{gathered}[/tex]

Therefore, P(A or B) =0.90

WILL GIVE BRAINLYEST 100 POINTS !!!! A student is painting a brick for his teacher to use as a doorstop in the classroom. He is only painting the front of the brick. The vertices of the face are (−6, 2), (−6, −7), (6, 2), and (6, −7). What is the area, in square inches, of the painted face of the brick?

144 in2
108 in2
72 in2
42 in2

Answers

The area, in square inches, of the painted face of the brick is; 144 in²

How to find the area of a square with coordinates?

The area of a square is given by;

A = L²

where;

L is the length of the side of the square

The sides of a square all have the same length, and as such we just need to find the length of one side.

The length of the side of the square here is the distance between two vertices, which can be calculated as

L = √[(x₂ - x₁)² + (y₂ - y₁)²]

However, to avoid long process, since it is a square, we can use subtraction of coordinates to get the side length which is gotten by using the first 3 coordinates;

Horizontal length = (6 + 6) = 12

Thus;

Area = L² = 12² = 144 in²

Read more about area of square coordinates at; https://brainly.com/question/14698897

#SPJ1

3. *Which of the following equations has x intercepts at 4 and -2? (A) y = 3x^2 - 10x - 8 (B) y = x^2 + 2x - 8 (C) y = 3x^2 – 2x – 8 (D) y = x^2 - 2x - 8

Answers

Given data:

The x-intercepts given are 4 and -2.

Substitute 0 for y in the first option.

[tex]\begin{gathered} 0=3x^2-10x-8 \\ 3x^2-12x+2x-8=0 \\ 3x(x-4)+2(x-4)=0 \\ x=4,\text{ -}\frac{2}{3} \end{gathered}[/tex]

Substitute 0 for y in the second option.

[tex]\begin{gathered} 0=x^2+2x-8 \\ x^2+2x-8=0 \\ x^2+4x-2x-8=0 \\ x(x+4)-2(x+4)=0 \\ (x-2)(x+4)=0 \\ x=2,\text{ -4} \end{gathered}[/tex]

Substitute 0 for y in the third option.

[tex]\begin{gathered} 0=3x^2-2x-8 \\ 3x^2-2x-8=0 \\ 3x^2-6x+4x-8=0 \\ 3x(x-2)+4(x-2)=0 \\ (x-2)(3x+4)=0 \\ x=2,\text{ -}\frac{4}{3} \end{gathered}[/tex]

Substitute 0 for y inlast option.

[tex]\begin{gathered} 0=x^2-2x-8 \\ x^2-4x+2x-8=0 \\ x(x-4)+2(x-4)=0 \\ (x-4)(x+2)=0 \\ x=4,\text{ -2} \end{gathered}[/tex]

Thus, option (D) is correct.

find the constamt of proportionality (r) in the equation y=rx

Answers

To find the constant of proportionality of the equation y = rx, from the values of the given table, just calculate the quotient r = y/x, where x and y can be any pair of values of the table.

For x = 3 and y = 30:

r = 30/3

r = 10

Hence, the constant of proportionality is

r = 10

What is the product of the complex numbers below?(3 - 2i)(1 + 7i)A.-11 + 19iB.17 + 19iC.-11 - 23iD.17 - 23i

Answers

Solution:

Given:

[tex](3-2i)(1+7i)[/tex]

To find the product, we multiply the terms in the second parentheses by each term in the first parentheses.

Thus, we have

[tex]\begin{gathered} 3(1+7i)-2i(1+7i) \\ open\text{ parentheses,} \\ 3+21i-2i-14i^2 \\ but\text{ i}^2=-1 \\ thus,\text{ we have} \\ 3+21i-2i-14(-1) \\ collect\text{ like terms,} \\ (3+14)+i(21-2) \\ \Rightarrow17+19i \end{gathered}[/tex]

Hence, the product of the complex numbers is

[tex]17+19i[/tex]

The correct option is B

Other Questions
To change a fraction into a decimal do you, divide the denominator by the numerator. A local bakery has determined the probability distribution for the number of cheesecake that they sell in a given day let X equal the number of cheesecake sold on a randomly selected day what is one important difference between a normal distribution and a t distribution? group of answer choices the t distribution has heavier tails than a normal distribution. the normal distribution has heavier tails than a t distribution. the two distributions have similar tails, it's just that the t distribution can be skewed, while the normal distribution may not be skewed. You have 26 video games in your collection.Some are for the X-Box system and someare for PlayStation. You have 4 fewerPlaystation games than X-Box games.Write a variable expression to model thesituation. How many of each type of gamedo you have? \Which of the following should be broken into ions when writing a complete ionic equation? Select all that apply. Which set of ordered pairs does not represent a function?O {(6, 7), (-4,-5), (-1, 7), (-9, -7)}O{(-6, 1), (-9, -6), (6, 7), (1, -8)}O {(5,9), (7,2), (1,6), (-6,9)}O {(-1, 1), (4, -6), (-3,0), (-3, 8)} Select all of the expressions that are less than 10103O A 103B. 1x 10oC 103 x 2OD } x 103O E 103 When an ice cube melts in your hand, it changes from a solid to a liquid.H2O (s) H2O (l)This is an example of ________________ reaction, because heat is _____________ the system. 2. Connect an ammeter and voltmeter in the circuit below. A data set has these values: 6, 8, 8, 10, 10, 10, 10, 12, 12, 14. The histogram ofthe distribution is shown.Aanb542 A local produce stand has eight pumpkins on display the diameters of the pumpkin in inches are given in the table Choose the best Spanish equivalent to the phrase.We are fourteen years old.Somos catorce aos viejo.Estamos catorce aos viejo.Tenemos catorce aos.Hacemos catorce aos. I need the answer! I snap a pic to try n get it but no answer lol someone help!!! Im on the clock What sentence best supports the staternent that hormones are involved in the regulation of homeostasas? The hormone melatonin induces sleep and its production is slowed by exposure to light. The hormone cortisol suppresses the immune system and is produced when the body is under stress. The hormone oxytocin promotes labor contractions of the uterus during childbirth. The hormone erythropoeitin increases the production of red blood cells when oxygen levels are low. Use my radians find the amplitude and period of each function then graph Inequality statement for -13,-25,-8 After the Civil War, federal troops remained in the South for 12 years. During this period, known as Reconstruction, the federal government made a concerted effort to improve the legal and social status of formerly enslaved Black people. The troops were there in part to enforce new laws including the 13th, 14th, and 15th Amendments. The 13th Amendment outlawed slavery, the 14th promised all Americans equal protection under the law, and the 15th granted Black men voting rights. But in 1877, federal troops left the South as part of a compromise in a contested presidential election. Reconstruction came to an abrupt end. As Southern Democrats returned to power, they enacted Jim Crow laws aimed at reversing progress toward racial equality. These state and local laws prevented African Americans from voting and segregated public accommodations like schools, waiting rooms, and transportation.According to the passage, which of the following events led to the end of Reconstruction?AThe final battle of the Civil WarBThe ratification of the 13th, 14th, and 15th AmendmentCThe integration of public accommodationsDThe removal of federal troops from the South 2.12.1: LAB: Name formatThis is what I have so far:name_input = input()name_separator = name_input.split()if len(name_separator) == 3: first_name = name_separator[-3] middle_name = name_separator[-2] last_name = name_separator[-1] first_initial = first_name[0] middle_initial = middle_name[0] last_initial = last_name[0] print(last_name + ", " + first_initial + '.' + middle_initial +'.')elif len(name_separator) == 2: first_name = name_separator[-2] last_name = name_separator [-1] first_initial = first_name[0] last_initial = last_name[0] print(last_name + ", " + first_initial + ".") Shane bought 36 gallons of gas this week this amount is 6 gallons more than twice the amount of gas he bought last week how many gallons of gas in Shane by last week PLEASE HELP FAST!!!!!!! Some information appears for you. Amendment Process: Two Main Steps 1: ____________________ 2: RatificationMethod 1 Method 2 Two-thirds of state legislatures request a national convention